economics - prepmate

71

Upload: others

Post on 12-Dec-2021

11 views

Category:

Documents


0 download

TRANSCRIPT

Page 1: Economics - PrepMate
Page 2: Economics - PrepMate

Australia • Brazil • India • Mexico • Singapore • United Kingdom • United States

Economicsfor

UPSC and State Civil Services

Examinations

Page 3: Economics - PrepMate

© 2019 Cengage Learning India Pvt. Ltd.

ALL RIGHTS RESERVED. No part of this work covered by the copyright herein may be

reproduced, transmitted, stored, or used in any form or by any means graphic, electronic,

or mechanical, including but not limited to photocopying, recording, scanning, digitizing,

taping, Web distribution, information networks, or information storage and retrieval systems,

without the prior written permission of the publisher.

For permission to use material from this text or product, submit all

requests online at

www.cengage.com/permissions

Further permission questions can be emailed to

[email protected]

ISBN-13: 978-93-86668-79-0

ISBN-10: 93-86668-79-3

Cengage Learning India Pvt. Ltd.

418, F.I.E., Patparganj

Delhi 110092

Cengage Learning is a leading provider of customized learning solutions with office locations

around the globe, including Australia, Brazil, India, Mexico, Singapore, United Kingdom and

United States. Locate your local office at: www.cengage.com/global

Cengage Learning products are represented in Canada by Nelson Education, Ltd.

For product information, visit www.cengage.co.in

Economics

Page 4: Economics - PrepMate

Preface xix

Acknowledgements xxi

List of Videos xxiii

Chapter-wise Break up of Previous Year’s Questions (Prelims) xxiv

1 ECONOMIC GROWTH AND DEVELOPMENT 11 Gross Domestic Product 1

What Does GDP Indicate? 1Criticism of GDP 1Calculation of GDP 1India’s Position (February 2018) 2What is Purchasing Power Parity 2Difference between Nominal GDP and Real GDP 2GDP Growth 3

2 Market Price and Factor Cost 4What Is Factor Cost? 4What Is Market Price? 4Relationship between GDP at Market Price and GDP at Factor Cost 5

3 Gross Value Added or GDP (FC) 5Relationship to GDP 5Why GVA Is Calculated? 5Use of GVA as Primary Measure of Economic Activity? 5What is the Global Trend? 5

4 Gross National Product 6How Is GNP Calculated? 6Comparison with GDP 6

5 Net Domestic Product (at Market Price) 6How Do We Calculate NDP? 6

6 National Income or Net National Product at Factor Cost 67 Per Capita Income 78 Which Is a Better Measure of Economic Growth: GDP or Per Capita income? 79 Definition of Economic Development 9

Key Differences between Economic Growth and Economic Development 1010 Inclusive Growth 10

Approaches to Attain Inclusive Growth 10

CONTENTS

Page 5: Economics - PrepMate

iv Contents

11 Lorenz Curve 11Gini Coefficient 11Lorenz Curve, Gini Coefficient, and Measuring Inequality 11

12 Terms Denoting Level of Economic Activity 1213 Poverty Estimates in India 12

Tendulkar Committee, 2005 (Headed by Suresh Tendulkar) 13Rangarajan Committee 13

14 Unemployment 14Types of Unemployment 14Calculation of Unemployment 15

15 Phillips Curve 1516 Human Resource Development 1617 Human Development Index 16

New Method for Calculating HDI 16Value of HDI 17

18 Multidimensional Poverty Index 17Indicators 18Calculation of the Index 18

Practice Questions 19Perfecting Past Prelims 25

2 TAXATION SYSTEM OF INDIA 291 Regressive Taxes 292 Proportional Taxes 293 Progressive Taxes 294 Direct and Indirect Taxes 29

Conclusion 305 Important Taxes in India 31

Income Tax 31Excise Duty 33Customs Duty 33Service Tax 33Sales Tax or VAT 33

6 Goods and Services Tax (GST): Change in Indirect Taxation System 34Components of GST: SGST, CGST and IGST 35Composition of the GST Council 35GST Rate 35Commodities Outside GST 36Compensation to States 36National Anti-Profiteering Authority 36e-way Bill 37Implications of Introducing GST 38Administration of GST 38

Page 6: Economics - PrepMate

vContents

Input Credit Under GST 39Conditions for Claiming Input Credit 39Challenges in Introduction of GST 40

7 Tax-to-GDP Ratio 418 Laffer Curve 419 Minimum Alternate Tax 42

Why Does It Arise? 42What Are the Differences in These Two Incomes? 42

Practice Questions 44Perfecting Past Prelims 47

3 MONETARY POLICY 511 Monetary Policy 51

Relation between Inflation and Economic Growth 51Working of Monetary Policy 51

2 Tools to Exercise Monetary Policy 51Quantitative Measures 52Qualitative Measures 55

3 Marginal Cost of Funds Based Lending Rate versus Base Rate 56Marginal Cost of Funds Based Lending Rate 56Base Rate 56

4 Reserve Bank of India 57Eight Major Functions of the Reserve Bank of India 57

5 Types of Bank Accounts 586 What is Money Supply? 59

Calculation of Money Supply in India 59What Is the Relevance of Reserve Money? 60

Practice Questions 61Perfecting Past Prelims 66

4 FISCAL POLICY 711 Fiscal policy 71

High Expenditure and Low Revenue Collection 71Low Expenditure and High Revenue Collection 71

2 Exercising Fiscal Policy 71When Economic Growth Needs to Be Increased 72When the Economy Needs to Be Curbed 72

3 Whom Does the Fiscal Policy Affect? 724 Measures of Government Deficit 72

Revenue Deficit 73Fiscal Deficit 74

Page 7: Economics - PrepMate

vi Contents

Primary Deficit 74Effective Revenue Deficit 74

5 Deficit Financing 75Consequences of Deficit Financing 75Should Deficit Financing Be Resorted to? 76

6 Fiscal Responsibility and Budget Management (FRBM) Act, 2003 76Impact of Global Financial Crisis on FRBM Act, 2003 76Criticism of the FRBM Act 76

7 Budget 77Which Government Body Makes National Income Estimates in India? 77Economic Survey 78Zero-base Budgeting 78Performance Budgeting and Outcome Budgeting 79Gender Budgeting 79Performance Monitoring and Evaluation System 79Budget Break Up 80Budget Reforms 82

Practice Questions 84Perfecting Past Prelims 87

5 INFLATION 911 Types of Inflation 912 Effects of Inflation on Various Classes of Persons 933 Effects of Inflation on Various Economic Phenomenon 944 Factors Affecting Inflation 94

Role of RBI in Curtailing Inflation 945 Inflation in Different Types of Products 946 Inflation in Developed and Developing Countries 957 Measurement of Inflation 95

Inflation Indices 95Calculation of Inflation 97

8 Measures to Control inflation 97Practice Questions 98Perfecting Past Prelims 101

6 CLASSIFICATION OF INDIAN ECONOMY 1051 Sectors of Economy 1052 Goods and Services 1063 Comparison of Indian Economy with Economies of the World 1084 Usual Transition in Economy 1095 Peculiar Case of India’s Growth Story: Services-Led Growth 110

Consequences of Low Growth in Secondary Sector 111

Page 8: Economics - PrepMate

viiContents

6 Organized and Unorganized Sectors 111Indian Context 112

Practice Questions 114Perfecting Past Prelims 116

7 SUBSIDIES 1191 Subsidy 119

Direct versus Indirect Subsidies 1192 Subsidies in India 119

Food Subsidy 120Fertilizer Subsidy 121Fuel Subsidy 123

3 Subsidies for Agriculture 125Issues Related to Water Subsidy 125

4 Public Distribution System 126National Food Security Act, 2013 126Why PDS Reforms Are Required? 127

Practice Questions 128Perfecting Past Prelims 130

8 AADHAAR 1341 Aadhaar Act 1342 Present position on Aadhaar Link Program 1353 Supreme Court judgement on Aadhaar link Program 135

Rationale behind Supreme Court judgement 136Why Section 7 Benefits Are Treated in Different Manner? 136Whether Aadhaar Infringes Privacy of the Person? 136

4 Concerns over Aadhaar Project 136Reasons for Concerns Over Protection of Data 136Solutions to Protect Aadhaar Data 136Can Face Authentication Be Faked? 137

Practice Questions 137Perfecting Past Prelims 138

9 FINANCIAL MARKET 1401 Money Market 1402 Capital Market 1403 Primary Market and Secondary Market 143

Regulation of Exchanges 144List of Approved Stock Exchanges in India 144

Page 9: Economics - PrepMate

viii Contents

4 Derivatives 1455 Mutual Fund 147

Load or No-Load Funds 147Closed-End Funds 148Open-End Funds 148

6 Pension Fund 1487 Terms Related to Stock Exchange 1498 Company 149

Types of Companies 1499 Financial Sector Regulators 150

Competition Commission of India 150Insurance Regulatory Development Agency 150Pension Fund Regulatory Development Authority 150Financial Stability and Development Council 150

10 Credit Rating Agencies 151Big Three Rating Agencies: Moody’s Investors Service, Standard & Poor’s (S&P), and Fitch 151India’s Position in Leading Credit Ratings 151

Practice Questions 153Perfecting Past Prelims 157

10 POPULATION 1591 Theory of Demographic Transition 159

Stage I: High Death Rate and High Birth Rate 159Stage II: Low Death Rate and High Birth Rate 159Stage III: Low Death Rate and Low Birth Rate 161

2 Density of Population 1613 Urbanization and Economic Development 1634 Why Population Growth Inhibits Economic Development? 1635 National Population Policy 1656 Approach to Family Planning 1667 China’s Policy to Reduce Population 1668 United Nations Population Projections 166

Practice Questions 167

Perfecting Past Prelims 168

11 INDIAN AGRICULTURE I: MARKETS, IRRIGATION, AND FINANCE 1691 Agricultural Price Market Committee Act 169

Objectives of Mandi System 169Problems with Old APMC Act 169

2 Model APMC Act 170Salient Features of Model APMC Act 170

Page 10: Economics - PrepMate

ixContents

Additional Suggestions to Reform Model APMC Act 1713 Types of Irrigation Techniques 172

Surface Irrigation 172Localized Irrigation 172Sub-irrigation 173

4 Pattern of Irrigation in India 1735 Agricultural Credit in India 174

Types of Agriculture Credit 174Source of Agricultural Credit in India 174Trends in Agricultural Credit 176NABARD Initiatives for Rural Development 177Weaknesses in Rural Credit Structure 177

Practice Questions 178

Perfecting Past Prelims 180

12 INDIAN AGRICULTURE II: LAND REFORMS 183

1 Zamindari or Permanent Settlement System 1832 Ryotwari System 1833 Mahalwari System 1834 Land Reforms 184

Abolition of Intermediaries 184Tenancy Reforms 185Reorganization of Ownership of Land 186

5 Cooperative Farming 188Critical Evaluation of Cooperative Farming in India 188

Practice Questions 189

13 PLANNING 1911 History 191

Bombay Plan 1912 Five-year Plans in India 191

First Five-Year Plan (1951–56) 191Second Five-Year Plan (1956–61) 191Third Five-Year Plan (1961–66) 192Three Annual Plans (1966–69) 192Fourth Five-Year Plan (1969–74) 192Fifth Five-Year Plan (1974–79) 192Sixth Five-Year Plan (1980–85) 192Seventh Five-Year Plan (1985–90) 192Two Annual Plans (1990–92) 193Eighth Five-Year Plan (1992–97) 193

Page 11: Economics - PrepMate

x Contents

Ninth Five-Year Plan (1997–2002) 193Tenth Five-Year Plan (2002–07) 193Eleventh Five-Year Plan (2007–12) 194Twelfth Five-Year Plan (2012–17) 194

3 Types of Planning 194Single-Level Versus Multilevel Planning 194Decentralized Planning 195Difference between Multilevel Planning and Decentralized Planning 196

4 Criticism of Planning 1965 Imperative to Indicative Planning 1966 Future of Planning 1967 Planning Commission and NITI Aayog 197

Planning Commission 197NITI Aayog 197Differences between Planning Commission and NITI Aayog 198

Practice Questions 198

Perfecting Past Prelims 200

14 INDUSTRIAL POLICY 2031 Prior to 1991 203

Industrial Policy Resolution of 1948 203Industries (Development and Regulation) Act, 1951 203Rationale Behind Pre-1991 Industrial Policy 204Criticism 204Movement Towards Liberalization 204

2 Economic Reforms: Liberalization, Privatization, and Globalization 205Liberalization 205Privatization 205Globalization 205

3 Socialist versus Capitalist Economy 207Socialist Economies 207Capitalist Economies 208Relation between Liberalization, Privatization, and Globalization 209

4 Industrial Policy Since 1991 209Provisions of Industrial Policy 1991 209Policy for Medium-, Small-, and Micro-Scale Industry (Small-Scale Industry) 209Policy for Foreign Investment 210Disinvestment by the Government 210

5 Medium, Small, and Micro Industry 210Contribution of MSMEs in India 212Challenges 212Benefits of an MSME Registration 212

Page 12: Economics - PrepMate

xiContents

6 Trend in Indian Economy 2127 Indian Economy at Present 213

Entry of Private Sector 213Special Economic Zones 213National Manufacturing and Investment Zones 215

8 Food-Processing Industry: An Introduction 216Scope for High Growth 216Importance of Food-Processing Industry 216Challenges 217Mega Food Parks Scheme 217Government Initiatives 218

Practice Questions 221

Perfecting Past Prelims 223

15 INFRASTRUCTURE 2271 Transportation 227

Pattern of Development 227Problems in Development of Transportation System in India 227

2 Railways 228Profitability of Railways 228Growth of Railways Since Independence 230Advantages of Road Transport Over Railways 230Should Railways Be Privatized? 230Issue of Track Gauge in India 230Project Unigauge 231

3 Road Network 231National Highways 232State Highways and District Roads 232Rural Roads 232Shortcomings of Road Transport 232State Government Presence in Bus Transport 232

4 Indian Shipping 233Problems of Indian Shipping 233Development of Ports 233State of Coastal Shipping in India 234

5 Civil Aviation 234Reasons of Introduction of FDI in Civil Aviation up to 49% 235

6 Inland Waterways 2357 Power 235

High Dependence on Imports in Energy Sector 236Quality of Coal 237

Page 13: Economics - PrepMate

xii Contents

Opening up the Coal Sector to Private Players 237Crude Oil 238

8 Public–Private Partnerships 238Practice Questions 240

16 NON-PERFORMING ASSETS 2431 Definition of Non-Performing Assets 2432 Criteria Laid by RBI for NPA in Different Cases 2433 Categories of NPAs 2434 How Serious Is India’s NPA Issue? 244

Reasons for High NPAs 244Impact of High NPAs 244

5 Steps Taken to Tackle NPAs 245Debt Recovery Tribunal 245Securitisation and Reconstruction of Financial Assets and Enforcement of Security Interest (SARFAESI) Act, 2002 245Asset Reconstruction Company 245Bad Banks 246Emphasis on Debt Restructuring 246Mission Indradhanush for Public-Sector Banks 246Banks Board Bureau 246

6 Anchor Banks 247SBI Merger 247

7 Insolvency and Bankruptcy Code, 2016 2488 Basel III (Third Basel Accord) 2499 Securitization 24910 Letter Of Undertaking (Lou) And Pnb Fraud 249

What is a LoU? 249Punjab National Bank Fraud 250

Practice Questions 251Perfecting Past Prelims 253

17 FINANCIAL INCLUSION 2551 Need for Financial Inclusion 2552 Prerequisite for Financial Inclusion 255

Financial Literacy 255Availability of Financial Services at Affordable Cost 256

3 Mudra Bank 2564 Sharia/Islamic Banking 257

How Does Islamic Banking Work? 257Significance of Islamic Banking 257

Page 14: Economics - PrepMate

xiiiContents

5 Payments Bank 258How Is a Payments Bank Different from a Bank? 258How Payments Bank Will Ensure Financial Inclusion? 258Which Companies Have Been Given Approval? 258

6 Wholesale Banks 2597 Small Finance Banks 2598 Universal Banks 2609 Non-Banking Financial Companies 260

Popular NBFCs in India 261Merits of NBFCs 263Demerits of NBFCs 263

10 Self-Help Groups 263Objectives of SHGs 263

11 Microcredit 263Status of SHGs and Microfinance Movement in India 263Channels of Microfinance in India 264Concerns Over Microfinance Movement in India 265

12 Virtual Currency 265What Is Cryptocurrency? 265Difference Between Digital Currency and Virtual Currency 266Benefits of Virtual Currency 266Possible Threats after Widespread Use of Virtual Currency 266Initial Coin Offerings (ICO) 267Legal Status of Bitcoin 267Block Chain Technology 268

13 Unified Payments Interface (Upi) 270BHIM 270National Payments Corporation of India (NPCI) 271

Practice Questions 271Perfecting Past Prelims 273

18 PARALLEL ECONOMY 2771 Tax Planning 2772 Tax Avoidance 2773 Tax Evasion 2774 Black Market 277

Factors Supporting Black Markets 278Black Market Prices 278

5 Grey Market 2786 Black Economy (or Parallel Economy) 2787 Money Laundering 279

Some Common Methods of Money Laundering 279Challenges in Money Laundering 279

Page 15: Economics - PrepMate

xiv Contents

International Mechanism to Deal with Money Laundering 279FATF Lists 280Domestic Mechanism to Deal with Money Laundering 280

8 Hawala Transactions 2819 Base Erosion and Profit Shifting 28310 Tax Havens 28311 General Anti-Avoidance Rules 284

Why GAAR Were Discussed for the First Time? 284Further Issues While Implementing GAAR 284

12 Shell Companies 284Practice Questions 286Perfecting Past Prelims 287

19 REGIONAL TRADE AGREEMENT 2891 Preferential Trade Agreement 2892 Free Trade Agreement 289

CECA versus CEPA 2893 Customs Union 2904 Common Market 2905 Economic Union 2906 Economic Integration 2907 Impacts of Regional Trade Agreement 290

Impact on Production 291Impact on Consumption 291

8 Impacts of RTAs Among Other Nations Such As European Union on India 292Benefits to India 292Challenges 292

9 Problems Faced by India on Entering into RTAs 29210 Determinants of Success of RTAs 29311 Do RTAs Facilitate Multilateral Free Trade? 293

RTAs Facilitate Multilateral Free Trade 293RTAs Do Not Facilitate Multilateral Free Trade 293

Practice Questions 295

20 TRADE BARRIERS IN INTERNATIONAL TRADE 2971 Why Tariff and Non-Tariff Barriers Are Used? 2972 Custom Duties 298

Type of Custom Duties 2983 Non-Tariff Barriers 299

Traditional Barriers 299Non-Traditional Barriers 300

Practice Questions 301

Page 16: Economics - PrepMate

xvContents

21 FOREIGN EXCHANGE RATE SYSTEM OF INDIA 3031 1947–71 303

End of Bretton Woods Agreement 304

2 1971–75 304

3 1975–92 304

4 1992–93 304

5 1993 Onwards 304

Fixed Exchange Rate System Versus Market-Determined Exchange Rate System 305

6 Depreciation and Appreciation 305

Causes of Depreciation in Rupee 306

Causes of Appreciation of Rupee 306

Implications of Depreciation and Appreciation of Rupee 307

So Which Is Better: Depreciation or Appreciation? 307

7 How Is Devaluation Different from Depreciation? 308

When Does Government Decide to Devalue Its Currency? 308

Elasticity of Demand and Supply 308

Indian Foreign Trade in 1966 and 1991 309

8 NEER and REER 309

9 RBI Intervention in Foreign Exchange Market 309

10 Dirty Floating 310

11 China’s Currency System 310

Undervaluation of Chinese Currency 310

12 Implications of Common Currency Adoption by the European Union on India 310

13 Indicator of Sufficiency of Forex: Import Cover 311

Practice Questions 311

Perfecting Past Prelims 313

22 CONVERTIBILITY 3151 Current Account Convertibility 3152 Capital Account Convertibility 315

Probable Merits of Capital Account Convertibility 315Concerns Regarding Capital Account Convertibility 315Prerequisites for Introduction of Capital Account Convertibility 316Committee on Capital Account Convertibility 316

3 Foreign Exchange Regulation Act (FERA), 1973, and Foreign Exchange Management Act (FEMA), 1999 316

Practice Questions 317Perfecting Past Prelims 318

Page 17: Economics - PrepMate

xvi Contents

23 BALANCE OF PAYMENTS 3211 Current account transactions 3212 Capital account transactions 322

Implications of Current Account Deficit and Capital Account Surplus 3273 India’s Foreign Reserves 327

Foreign Exchange Reserves 3284 Use of Forex 328

Sovereign Wealth Funds 329Practice Questions 330Perfecting Past Prelims 331

24 FOREIGN INVESTMENT 3331 New Definitions of FDI and FPI 3332 Administrative Mechanism for Entry of FDI 333

Automatic Route 333Approval Route 334Foreign Investment Promotion Board 334

3 Ownership Share and Rights of Foreign Investor 3344 Why There Are Restrictions in Some Sectors? 3355 Prohibited Sectors 3386 Analysis of FDI Limit in Some of the Important Sectors 338

100% FDI in Defence 338FDI in Retail Sector 338FDI Reforms in Pharmaceutical Sector 339FDI in Airline Sector 339

7 India’s Share in Global FDI 339Regions Receiving Bulk of FDI 340Nation-wise FDI to India (2015–16) 340Double Taxation Avoidance Agreement 340

8 Foreign Portfolio Investment 341Issues Related to FPIs 341

9 Depository Receipt 342American Depository Receipt 342Global Depository Receipt 342Indian Depository Receipt 342

Practice Questions 343Perfecting Past Prelims 344

Page 18: Economics - PrepMate

xviiContents

25 INTERNATIONAL ECONOMIC ORGANIZATIONS 3471 International Monetary Fund 347

Sources of Fund 347India’s Relation with IMF 347Technical Assistance from IMF 348Role of IMF 348Criticism of IMF 348

2 World Bank Group 349International Bank for Reconstruction and Development 350International Development Association 350World Bank’s Assistance to India 350International Finance Corporation 351Multilateral Investment Guarantee Agency 351International Council for Settlement of Investment Disputes 352Criticism of World Bank 352Reports Published By IMF and World Bank 352

3 Asian Development Bank 3544 Asian Infrastructure Investment Bank 3545 New Development Bank 3546 European Bank For Reconstruction And Development (Ebrd) 355

EBRD and India 3557 Contingency Reserve Arrangement 3558 United Nations Conference on Trade and Development 356Practice Questions 356Perfecting Past Prelims 358

26 WORLD TRADE ORGANIZATION 3611 Broad Guidelines to Attain Objectives 3612 History 3623 Activities 362

4 Dispute Settlement 363

5 Agreements under WTO 363

Agreement on Goods 363

Agreement on Agriculture 363

6 General Agreement on Trade in Services 364

7 Intellectual Property and TRIPS 365

Patent 366

Geographical Indication 366

Page 19: Economics - PrepMate

xviii Contents

Copyright 367

Trademarks 367

Trade Secrets 368

8 Indian Patent Law 368

9 TRIPS and Agriculture 370

10 Trade-Related Investment Measures 371

11 Ministerial Conference 371

Outcomes of Important Meetings 372

11th WTO Ministerial Conference: Buenos Aires, Argentina, 2017 373

12 Trade Facilitation Agreement 374

13 Trade Facilitation in Services 374

14 National Food Security Act, 2013, and AOA 374

Practice Questions 375

Perfecting Past Prelims 377

27 INDIA’S TRADE POLICY 379

1 1947 Till Mid-1970s 379

2 Mid-1970s to 1990–91 379

3 Policy Since 1991 380

4 Export Promotion Measures 380

Bodies for Promotion of Exports 380

Schemes for Promotion of Exports 380

5 Foreign Trade 381

Exports 381

Imports 381

Crude Oil and Non-Oil Imports 382

Exports and Imports 382

6 India’s Largest Trading Partners 383

Practice Questions 383

Perfecting Past Prelims 385

28 MISCELLANEOUS TOPICS 3871 Demonetization 387

What Were the Expectations? 387Criticism 387Why Demonetization Was a Colossal Failure? 388Demonetization: A Legal Angle 388

Page 20: Economics - PrepMate

xixContents

2 Global Financial Crisis of 2008 389What Was the Reason behind the Global Financial Crisis? 389Measures Taken to Overcome Global Financial Crisis 389

3 Euro Zone Crisis 390Concerns over Debt Repayment by Government of Greece 390Impact of Euro Zone Crisis on Indian Economy 390

Solutions for Practice Questions and Perfecting Past Prelims 391

Introduction to Writing Answers for Mains 429

Previous Year’s Questions (Mains) with Solutions 437

Appendix I 481

Appendix II 561

Page 21: Economics - PrepMate
Page 22: Economics - PrepMate

PREFACE

If you ever happen to be walking down the streets of places where preparation for Civil Services is done, it will not be uncommon for you to come across or make the acquaintance of ‘several’ starry eyed yet completely committed IAS aspirants. Yet, ‘several’ would be an understatement given the number that runs into lakhs! But when we say committed, we mean it; these young men and women are ready to sacrifice almost all their youthful follows including sleep, comfort and even a semblance of a normal life to achieve one goal—IAS!

Sadly, this dream remains a distant one for a large majority of these aspirants in spite of the endless hours of study and sleep forsaken nights. When we tried to unravel WHY, the responses were almost synchronous:

“The subject was so vast that there was too much to cover and I could never complete it.”

“I read so much but could not retain it.”

“I studied something but was quizzed on something else in the exam.”

“I kept reading but did not attempt to solve the past year papers or give a mock exam.”

“Subscribing to several sources of information/preparation such as a coaching class, the internet and books was futile; after all there are only 24 hours in a day.”

“My almirah was full of too many books, but I could barely complete a few.”

And while the candid answers stated above clearly gave us a challenging problem—we did not attempt to solve it. We instead focused on a holistic solution—the synchronizing of effort i.e. Learning and Positive Results!

It is with this aim that we—PrepMate collaborated with Cengage India—are continuously striving to develop a comprehensive learning model that is a combination of online and offline so as to effectively address the issues that most aspirants grapple with.

About the Online–Offline Learning Model

The learning model initiates the process with a series of books targeted at cracking the UPSC exam. The books stand apart from others available because of the following unique features:

� We use a conceptual approach, simple language, explain concepts with diagrams, cite sufficient examples, pose pertinent questions in a reader friendly format—to ensure that the contents of these books can be read and assimilated in a time-bound manner.

� The content is specially designed taking into account the trend in UPSC exams in recent years. We have also included the previous years’ questions (with solutions) after every chapter.

Page 23: Economics - PrepMate

xxiiPreface

� The Practice Questions at the end of each chapter are exhaustive to provide sufficient preparation to crack the exams.

� The book series also contains additional information on ‘how to write answers’ along with what your approach should be for the mains—here too we have explained by solving questions and showing you the ‘preferred answering style’.

� We have tried to encapsulate all that is required to be learnt for a particular subject into a single book.

Usually, an aspirant purchases a book, but never gets a chance to contact the authors. We believe that the contact among aspirants and authors is important for learning and motivation of the aspirants. That is precisely why we have developed an application and a web portal to answer your queries and provide you with continuous support during your preparation.

It is through this online system that we provide the following services:

1. Videos covering important and difficult topics

2. Answer writing practice sessions

3. Daily prelims quiz

4. Assistance in interview preparation

5. Regular updates

6. Daily current affairs

7. Monthly current affairs magazine

8. Radio news analysis

9. Educational videos

10. Previous years’ papers and solutions

11. Free study materials

Looking forward to being your partner in the journey towards achieving your dream!

In case you have any specific queries or constructive feedback you can always share the same with us via e-mail at [email protected] .

PrepMate

Page 24: Economics - PrepMate

“We cannot accomplish all that we want to do without working together”

The complete UPSC learning module by Prepmate has been the culmination of more than a year of ideation and brain storming by a lot of people. It is only natural that we should gratefully acknowledge their valuable contribution sincerely. I, Shubham Singla, founder of PrepMate Edutech, thank you all for being with me in this whole project. Rajinder Paul Singla, Nirmal Singla, Ramnik Jindal, Sharat Gupta, Subhash Singla and Vijay Singla—thank you for your continuous support and motivation.

We would also like to thank Maninder Mann and Sundeep Singh Garha who helped us in first conceiving and later developing the synergistic print–digital model of the project— without you we would be missing our competitive edge.

Implementation of strategy can more often than not prove challenging and the development of the online module did prove to be tougher than we had envisaged. But our technical team was focused on enabling our dream and delivering the best, and they surely did. With a specific mention to the testing of both the website and the application, we would like to thank Surabhi Misra, Parth and Tanvir who did their job patiently and effectively in spite of the road blocks.

Our videos and books could not have been possible without the help of our graphics design team—Sandeep, Sukhjinder and Roshni toiled endlessly to ensure the best designed audio-visuals.

It is an understatement to state that the sourcing and reviewing of existing content and the generation of content was the most crucial part of this project and the backbone of our Learning Module. This would just not have been possible without our team of content contributors: Isha Gupta, Shelly Jindal, Gurdeep Kaur, Surabhi Misra, Shaffy Garg, Dipika Arora, Sunil, Bhupinderjit Singh, Shantnu, Tanvir, Anmol, Kriti, Tanya, Sahil, Suraj and Dilshad, who left no stone unturned in their pursuit of excellence—your pivotal contributions are gratefully acknowledged.

We would like to extend a special thanks to our staff members Geeta, Jitender, Manoj and Pinki, who helped us in the most laborious job i.e. typing through the several manuscripts of our books—your contribution is sincerely appreciated.

It is imperative that we thank Isha Gupta, Shelly Jindal, Anjum Diwan, Rajesh Goel, Shikha Sharma and Ravinder Indoura, for their critical yet constructive feedback that identified and subsequently rectified the errors that crept in during the development process. We will never be able to thank them enough for this—you fortified the very foundation of our model.

We sincerely acknowledge the initiatives and support from the entire editorial team of Cengage India in the process of publishing this book.

PrepMate

ACKNOWLEDGEMENTS

Page 25: Economics - PrepMate
Page 26: Economics - PrepMate

LIST OF VIDEOS

1. How to Prepare Economics for UPSC Prelims and Mains

2. Indicators of Economic Activity (GDP and Others)

3. Taxation System

4. GST

5. Monetary Policy

6. Fiscal Policy

7. Inflation

8. Foreign Exchange Rate System of India

9. Balance of Payments

10. World Trade Organization

Page 27: Economics - PrepMate

Ch

apte

r-w

ise

Bre

ak U

p o

f P

rev

iou

s Y

ear’

s Q

ues

tio

ns

(Pre

lim

s)

Ch

apte

r n

ames

2

01

82

01

72

01

62

01

52

01

42

01

32

01

22

01

12

01

02

00

92

00

8T

ota

l

1.

Eco

no

mic

gro

wth

an

d d

evel

op

men

t4

13

11

212

2.

Tax

atio

n s

yste

m1

12

12

11

110

3.

Mo

net

ary

po

licy

11

12

26

22

41

22

4.

Fis

cal p

oli

cy1

12

1

1

14

11

5.

Infl

atio

n2

13

22

10

6.

Cla

ssifi

cati

on

of

Ind

ian

eco

no

my

22

4

7.

Sub

sid

ies

21

1

15

8.

Aad

har

22

9.

Fin

anci

al M

ark

et1

1

13

6

10.

Po

pu

lati

on

11

13

11.

Ind

ian

Agr

icu

ltu

re I

: Mar

ket

s,

Irri

gati

on

an

d F

inan

ce

11

11

15

12.

Ind

ian

Agr

icu

ltu

re I

I: L

and

Ref

orm

s

13.

Pla

nn

ing

11

2 2

6

14.

Ind

ust

rial

Po

licy

11

11

31

8

15.

Infr

astr

uct

ure

16.

No

n P

erfo

rmin

g A

sset

s2

11

4

17.

Fin

anci

al I

ncl

usi

on

21

41

23

13

18.

Par

alle

l Eco

no

my

11

19.

Reg

ion

al T

rad

e A

gree

men

t

Page 28: Economics - PrepMate

Ch

apte

r n

ames

2

01

82

01

72

01

62

01

52

01

42

01

32

01

22

01

12

01

02

00

92

00

8T

ota

l

20.

Tra

de

Bar

rier

s in

In

tern

atio

nal

21.

Fo

reig

n E

xch

ange

rat

e sy

stem

of

Ind

ia1

11

14

22.

Co

nve

rtib

ilit

y1

1

23.

Bal

ance

of

Pay

men

ts1

21

4

24.

Fo

reig

n I

nve

stm

ent

11

11

4

25.

Inte

rnat

ion

al E

con

om

ic O

rgan

izat

ion

s2

2

4

26.

Wo

rld

Tra

de

Org

aniz

atio

n1

21

11

21

9

27.

Ind

ia’s

Tra

de

Po

licy

11

2

Ap

pen

dix

-I G

ove

rnm

ent

Sch

emes

211

32

32

12

26

T

ota

l 2

02

01

91

89

18

14

20

30

53

17

6

200

8

055

3

10

15

20

20

14

18

9

18

18

20

25

30

30

Nu

mb

er

of

qu

esti

on

s a

sked

un

der

Eco

no

mic

s S

ecti

on

35

2009

2010

20

11

2012

2013

201

42015

2016

2017

20

2018

Page 29: Economics - PrepMate
Page 30: Economics - PrepMate

Chapter 1 ECONOMIC GROWTH

AND DEVELOPMENT

1 GROSS DOMESTIC PRODUCT

Gross domestic product (GDP) is the monetary value of all the finished goods and services produced within a country’s borders in a specific time period. Though GDP is usually calculated on an annual basis, it can be calculated with reference to any time frame.

What Does GDP Indicate?

GDP is commonly used as an indicator of the economic health of a country, as well as a measure of country’s standard of living.

The higher the production of goods and services in an economy, the higher the consumption level of people. A higher consumption level indicates a higher standard of living of the people.

Criticism of GDP

1. Critics of GDP argue that it does not take into account the transactions that are illegal and not reported to the government in order to evade taxes.

2. Others criticize the tendency of GDP to be interpreted as an indicator of well-being, whereas in reality, it serves as a measure of a nation’s productivity. For example, people may not enjoy well-being on account of high pollution in their vicinity.

Calculation of GDP

There are three primary methods by which GDP can be determined. All, when correctly calculated, should yield the same figure. These three approaches are the expenditure approach, the output (or production) approach, and the income approach.

The expenditure approach measures the total value of all the products used in developing a finished product for sale to final consumers. For instance, a finished car’s contribution to a nation’s GDP would be measured by the total cost of materials and services that went into the car’s construction.

The production approach is something like the reverse of the expenditure approach. Instead of exclusively measuring the input costs that feed the economic activity, the production approach estimates the total value of final economic output to be consumed by ultimate consumers.

The third approach, the income approach, is an intermediary between the two above-mentioned approaches. It measures GDP by totalling all the domestic income earned by the private as well as government sectors.

Page 31: Economics - PrepMate

Economics2

India’s Position (February, 2018)

In India, GDP is estimated by the Central Statistics Office (CSO). The economy of India is the seventh largest (the United States, China, Japan, Germany, the United Kingdom, France, and India, in the order) economy in the world, measured by nominal GDP ($2.5 trillion) and the third largest (China, the United States, and India, in the order) by purchasing power parity (PPP) ($9.447 trillion).

What Is Purchasing Power Parity?

Purchasing power parity (PPP) is defined as the number of units of a country’s currency required to buy the same amount of goods and services in the domestic market as one dollar would buy in the US.

PPP is an attempt to work out how much currency will be needed to buy the same quantity of goods and services in different countries. It reflects the underlying exchange rate between the two different countries for buying goods and services, and a more accurate reflection of actual living standards in countries.

Often exchange rates do not actually reflect different living costs because some goods are not easily traded. For example, if you live in the United States, it is irrelevant if there is cheaper accommodation elsewhere in the world. What is relevant to you is the price of goods and services at which they are available to you locally.

For example, let us suppose that the market exchange rate between Dollar and Rupee is 66. One Dollar in the US buys 1 liter of milk. One dollar in terms of rupee, i.e., `66 can buy 3 liters of milk in India.

Suppose that India’s GDP is `660. If we consider the market exchange rate, then this amount is equal to $10. If milk is the only commodity produced in the world, one will think that India is producing 10 liters of milk considering India’s exchange rate `66 and GDP value of `660.

However, India produces 30 liters of milk. To overcome this defect, we use the purchasing power parity exchange rate. Under PPP, we measure the GDP of India by comparing the money required to purchase commodities in both the countries. One dollar in the US can purchase one liter of milk, whereas `21 can purchase one liter of milk in India.

$1 = `21Thus, in our example, PPP exchange rate is `21/$. Using this exchange rate, India’s GDP of `660

is equal to $30. Thus, India’s GDP in terms of PPP is $30. On the other hand, India’s GDP in terms of market exchange rate is $10.

The World Bank (WB) calculates the PPP exchange rate. As per the WB estimate in the year 2014,

`17.12 is equal to US$ 1.

Difference Between Nominal GDP and Real GDP

When the GDP is estimated at current prices, it exhibits Nominal GDP, whereas Real GDP is when the estimation is made at constant prices.

Definition of nominal GDP

It is the monetary value of the economic output produced during the current year at current year prices. The point to be noted is that the prices of the current year are taken into consideration without adjusting for inflation.

Current production of goods and services × Current prices = Nominal GDP

Page 32: Economics - PrepMate

Economic Growth and Development 3

Definition of real GDP

GDP measurement is done at fixed prices, i.e. at the prices which were prevalent at some point of time in the past, known as base year price or reference price. It reflects the economic output at constant prices. Real GDP is considered as a true indicator of country’s economic growth because it exclusively considers the rise in production of goods and services as the reason for increase in GDP.

Current Production of goods and services × Base Year Prices = Real GDPEconomists while calculating growth in GDP consider figures of real GDP.

GDP Growth

What is “economic growth rate”?

An economic growth rate is a measure of economic growth from one period to another in terms of percentage. In practice, it is a measure of the rate of change in a nation’s GDP from one year to another.

The economic growth rate provides insight into the general direction and magnitude of growth for the overall economy. It demonstrates the change in a nation’s or economy’s income over a specified period of time. Most commonly, this is examined on a quarterly basis, but economic growth rates can be observed across larger spans of time, such as year over year (YOY) or decade over decade.

Economic growth refers to positive change in economic output, but changes in economic output can be either positive or negative. If an economy experiences two consecutive quarters with falling growth rates, it can be said that the associated economy is falling into a recession. If the economy begins to shrink, the percentage rate can be expressed in negative to demonstrate the income lost over the time period being examined.

Real economic growth rate

The real economic growth rate measures the economic growth in relation to GDP, from one period to another, adjusted for inflation, in other words, expressed in real as opposed to nominal terms.

The real economic growth rate is expressed as a percentage that shows the rate of change of a country’s GDP from one period to another, typically from one year to the next.

The real economic growth rate, also referred to as the growth rate of real GDP, is a more useful measure than the nominal GDP growth rate due to the fact that it takes into account the growth only due to increase in output (and ignores the contribution of price rise to increase in value of output).

Calculating the real GDP growth rate

For instance, using expenditure method GDP is calculated as the sum of consumer spending, business

spending, government spending, and the total of exports minus imports. In order to factor in inflation

and arrive at the real GDP figure, the calculation is as follows: GDP Real GDP = __________________________ (1 + Inflation since base year)

Once real GDP is calculated, the real GDP growth rate is calculated as follows:

(Most recent year’s real GDP − Previous year’s real GDP)Real GDP growth rate = _____________________________________________________

¥ 100

Previous year’s real GDP

Base Year

The base year, updated periodically by the government, is a designated year used as a comparison point for economic data such as the GDP. Presently, 2011–12 is used as the base year for calculating GDP in India. Recently, the government has proposed to change the base year to 2017–18.

GDP is calculated by taking into account the quantity of the present year and prices of the base year. Thus, when we talk about GDP growth, we talk about real GDP growth rate.

Page 33: Economics - PrepMate

Economics4

GDP Growth Level

The real GDP growth is expected to reach 6.75 percent in 2017–18 and will rise to 7.0 to 7.5 percent in 2018–19, thereby re-instating India as the world’s fastest growing major economy. The GDP growth has averaged 7.3 percent for the period from 2014–15 to 2017–18.

GDP GROWTH

Source: Economic Survey 2017–18

7.5%6.5%**

8.0%7.1%*

At Constant Prices

*Provisional Estimates

2014

–15

2015

–16

2016

–17

2017–18

**1st Advance Estimates

GDP projections

for 2017–18

expected to

be 6.75%

2 MARKET PRICE AND FACTOR COST

What Is Factor Cost?

A number of inputs are required in the production process. These inputs are commonly known as factors of production and include things such as land, labour, capital, and entrepreneurship. Producers of goods and services incur a cost for using these factors of production. Factor cost refers to the cost of factors of production that is incurred by a firm when producing goods and services. Examples of such production costs include the cost of renting machines, salaries and wages, cost of obtaining capital, and the profit margins that are added by the entrepreneur (profits are the cost of entrepreneur).

The factor cost does not include the taxes that are paid to the government since taxes are not directly involved in the production process and, therefore, are not a part of the direct production cost. However, subsidies received are included in the factor cost as subsidies are given on inputs used in production.

What Is Market Price?

Once goods and services are produced, they are sold at a market price. The market price is the price that consumers will pay for the product when they purchase it from the sellers. Taxes charged by the government will be added onto the factor price while subsidies provided will be reduced from the factor price to arrive at the market price.

Taxes are added because taxes are the costs that increase the price, and subsidies are reduced because subsidies compensate the factor cost or reduce the factor cost. For instance, money is required to run a

Page 34: Economics - PrepMate

Economic Growth and Development 5

business. Thus, money is a factor of production. The cost of money is interest. Interest here is a factor cost. If government gives subsidy on interest, then actual cost of money (interest cost) will reduce.

Relationship between GDP at Market Price and GDP at Factor Cost

GDP (MP) – Indirect Taxes + Subsidies = GDP (FC)

� Note: In common parlance, when we use the term GDP, we refer to GDP at market prices.

3 GROSS VALUE ADDED OR GDP (FC)

Gross value added (GVA) is the measure of the value of goods and services produced in an area, industry, or sector of economy.

Relationship to GDP

GVA is linked to GDP, as both are measures of output. The relationship is defined as follows:

GVA + Taxes on products − Subsidies on products = GDP

Why GVA Is Calculated?

1. GVA and GDP give a picture of economic activity from producers’ (supply side) and consumers’ (demand side) perspective, respectively, because GVA is the net receipt of the producers and GDP is the expenditure incurred by the consumers.

2. Both these measures need not match and there could be a sharp divergence due to net indirect taxes (NIT = indirect taxes − subsidies), which are counted in GDP calculations (GDP is the sum of GVA and NIT).

3. GVA provides a better measure of economic activity because GDP can record a sharp increase just on account of increased tax collections due to better compliance/coverage and not necessarily due to increase in output.

4. GVA is a better reflection of the productivity of the producers as it excludes the indirect taxes, which could distort the production value.

5. A sector-wise breakdown provided by the GVA measure can better help policymakers to decide which sectors need incentives/stimulus or vice versa.

Use of GVA As Primary Measure of Economic Activity?

In the revision of the National Accounts Statistics in 2015 by the CSO, it was decided that GVA should

be adopted as primary measure of economic activity. In 2018, Reserve Bank switched back to the gross

domestic product (GDP)-based measure to offer its growth estimates, citing global best practices.

What Is the Global Trend?

Globally, the performance of most economies is gauged in terms of gross domestic product (GDP). This is also the approach followed by multilateral institutions, international analysts and investors, and primarily they all stick to GDP because it facilitates easy cross-country comparisons.

Page 35: Economics - PrepMate

Economics6

4 GROSS NATIONAL PRODUCT

Gross national product (GNP) refers to goods and services produced by Indians whether in India or abroad.

How Is GNP Calculated?

GNP = GDP − Production by foreigners in India + Production by Indians outside India

or

GNP = GDP − Factor income paid to foreigners in India + Factor income paid to Indians abroad

i.e., GNP = GDP − Net factor income from abroad

GNP refers to the monetary value of all the finished goods and services produced by nationals

(citizens) anywhere in the world in a specific time period.

Comparison with GDP

GDP is the monetary value of all the finished goods and services produced in a territory such as India,

whereas GNP is the monetary value of all the finished goods and services produced by nationals such

as Indians.

In India, the value of GDP is higher than that of GNP because India has received more investment

from abroad compared to the investment made by Indians in abroad.

5 NET DOMESTIC PRODUCT (AT MARKET PRICE)

Whenever there is production of goods and services, there is consumption or reduction in value of

assets used to produce goods and services; this consumption or reduction in value of assets is called

depreciation.

How Do We Calculate NDP?

Whenever we calculate the net value from gross value, we subtract depreciation out of gross value.

Net Domestic Product (NDP) = GDP − Depreciation

6 NATIONAL INCOME OR NET NATIONAL PRODUCT AT FACTOR COST

Net national product at factor cost (NNPfc) or national income is the income earned by nationals (such as Indians) in a given time period.

In other words, it is the collective income of nationals in a given time period. It is calculated as follows:

NNPfc = GNPmp – Depreciation – Indirect taxes + Subsidiesor NNPfc = GDPmp – Net factor income abroad – Depreciation − Indirect taxes + Subsidies

Page 36: Economics - PrepMate

Economic Growth and Development 7

7 PER CAPITA INCOME

Per capita income or average income measures the average income earned per person in a given area (city, region, country, etc.) in a specified year. It is calculated by dividing the area’s total income by its total population.

In other words, per capita income refers to national income divided by the population of a country.

Per capita income = National income

Population

India’s per capita income in 2015–16 at the current prices was `93,293, compared to `86,879 in the preceding year.

In real terms, the per capita income (at 2011–12 prices) during 2015–16 is estimated to have attained a level of `77,435, up 6.2% from `72,889 for 2014–15.

Real per capita income is obtained after adjusting nominal per capita income for inflation.

Real per capita income = Nominal per capita income / (1 + Inflation rate)

Let us understand the above concepts with actual figures given in the following table:

(`1000 crore)

Year Consumption of

fixed cap.

(depreciation)

Indirect

taxes less

subsides

GDP

at market

prices

NDP

at market

prices

Net factor

income

from

abroad

NNP at

factor cost

or net

national

income

Per capita

income (`)

2015–16 1278 923 11,350 10,072 −137 9012 77,435

2014–15 1193 825 10,552 9359 −124 8410 72,889

2013–14 1102 755 9839 8737 −122 7860 68,867

NDP at market prices = GDP at market prices – Depreciation

NNP at factor cost = NDP at market prices – Indirect taxes less subsidies – Net factor income abroad

Per capita income = Net national product at factor cost

Population

8 WHICH IS A BETTER MEASURE OF ECONOMIC GROWTH: GDP OR PER

CAPITA INCOME?

Economic growth is measured on the basis of expansion of GDP. However, there are instances when the rate of population growth is higher than the rate of increase in GDP. In such instances, GDP increases while per capita income decreases. Therefore, per capita income is considered a better indicator of economic growth.

Page 37: Economics - PrepMate

Economics8

Can Indian Economy Grow at 8-9% per annum?

The Indian economy is currently passing through a phase of relatively slow growth. However,

over the 9-year period beginning 2005–06, the average annual growth rate was 7.7%. Against

this background, the relevant question is whether India has the capability to grow at 8–9% in a

sustained way.

Past PerformanceIndia achieved a growth rate of 9.5% in 2005–06, followed by 9.6% and 9.3% in the subsequent 2

years. After declining a bit in the wake of international financial crisis, the growth rate went back to

8.9% in 2010–11. The domestic savings rate during this period averaged 34.9% of GDP. Similarly,

the gross capital formation rate averaged 36.2%.

Reasons for Reduction in Growth Rate: Low Investment and

High ICORAn analysis of the data of the period since 2012–13 reveals two trends. First, there has been a decline

in investment rate. Second, the decline in growth rate is greater than the decline in investment rate,

indicating a rise in the incremental capital–output ratio (ICOR). In 2007–08, India’s investment rate

was 38% of GDP. It declined steadily to touch 34.8% in 2012–13. The declining trend continued in

2015–16.

With an ICOR of 4, only a return to higher level of savings and investments can take us back to

8–9% growth seen earlier. Thus, what is needed to achieve the “higher growth rate” is to raise the

investment rate and improve the productivity (or use) of capital.

Incremental Capital–Output RatioICOR refers to addition in capital required to raise output by `1. The higher the ICOR, the lower

the productivity of capital. Thus, a high ICOR can be thought of as a measure of the inefficiency

with which capital is used. In most countries, the ICOR is near 3. ICOR, thus, determines efficiency

in use of capital. In India, ICOR is slightly high at 4. ICOR is low in service industry than in the

manufacturing sector.

It is influenced by a number of factors such as technology, skill of the labour force, which in

turn depends on the quality of the education system and ease of doing business, etc. Bureaucratic

hurdles, which impede speedy execution of projects, need to be removed. Thus, improving the

productivity of capital requires steps at several fronts.

What is “Savings Rate”?A savings rate is the amount of money, expressed as a percentage or ratio, that a person deducts from his or her income to set aside as savings. The accumulated savings are invested in various forms.

Page 38: Economics - PrepMate

Economic Growth and Development 9

Global Trends in “Savings Rate”For years, the savings rate in the United States has declined. In the 1970s and 1980s, personal savings rates were in the range of 5–7% but decreased to the range of 1–3% in the 21st century. In sharp contrast, the Chinese and Indian savings rate is about 30%.

The ratio of domestic saving to GDP reached from a peak of 38.3 percent in 2007 to 29.2 percent in 2013, before falling back to 29 percent in 2016.

The fall from 2007 to 2016 has been milder for investment than saving. In India, the investment slowdown started in 2012, subsequently intensified and was apparently still continuing as shown by 2016 figures.

What Affects the Savings Rate?The national average savings rate is often determined by how a particular culture views debt (loan), values possessions, etc. Economies oriented towards consumption have lower savings rates; in the United States, consumption constitutes around 75% of the economy. Economies such as India, which is oriented more towards investment, have higher savings rates. Savings rate tends to fall as the average age of the population increases. Savings rates are also affected by rise in income levels.

Calculating Savings RateThe savings rate is the ratio of personal savings to income and can be calculated for an economy as a whole or at the personal level. Savings

Saving rate = –––––––– × 100 Income

Investment RateThe investment rate refers to the proportion of GDP invested into economy. High savings rate leads to high investment rate in the economy. Economic growth depends on investment rate and ICOR. In other words, Economic growth = Investment rate × (1/ ICOR).

Let us presume that investment rate is 32% and ICOR is 4. Economic growth = 32% × (1/4) = 8%

9 DEFINITION OF ECONOMIC DEVELOPMENT

This approach emerged because overemphasis on quantitative goals such as increase in GDP levels

often made nations ignore the equally essential non-economic parameters such as health, education, life

expectancy, and quality of life.

Economic development is defined as the process of increase in volume of production along with the

improvement in technology, a rise in the standards of living, health, education, overall quality of life, etc.

To understand the two terms “economic growth” and “economic development”, we will take an example of

a human being. The term “growth of human beings” simply means increase in their height and weight, which

is purely physical. But if you talk about “human development”, it will take into account both the physical and

abstract aspects such as maturity level, attitudes, habits, behaviour, feelings, intelligence, and so on.

Page 39: Economics - PrepMate

Economics10

Similarly, the growth of an economy can be measured through the increase in its size in the current

year in comparison to previous years, but economic development includes not only physical but also

non-physical aspects that can only be experienced, such as improvement in the lifestyle of inhabitants,

increase in individual income, improvement in technology and infrastructure, etc.

Therefore, we can say that economic development is a much bigger concept than economic growth.

In other words, economic development includes economic growth. In context of India, we can say that

India is witnessing a period of fast economic growth but slow economic development.

Key Differences between Economic Growth and Economic Development

The fundamental differences between economic growth and development are explained in the following

points:

1. Economic growth is the positive change in the real output of the country in a particular span of

time. Economic development involves a rise in the level of production in an economy along with the

advancement of technology, improvement in living standards, and so on.

2. Economic growth is one of the essential prerequisites of economic development. However, it is not the

only requirement for economic development.

3. Economic growth enables an increase in the indicators such as GDP, per capita income, etc. On the other hand, economic development enables improvement in the life expectancy rate, infant mortality rate, literacy rate, and poverty rates.

4. Economic growth results in quantitative changes, but economic development brings both quantitative and qualitative changes.

5. Economic growth can be measured in a particular period. On the other hand, economic development is a continuous process, which can be seen in the long run.

10 INCLUSIVE GROWTH

Inclusive growth is that economic growth which benefits all the sections of society.

Inclusive growth basically means making sure everyone is included in growth, regardless of their

economic class, gender, sex, disability, and religion.

The inclusive growth approach takes on long-term perspective, and the focus is on productive

employment rather than a mere income redistribution among poor people.

With liberalization, the benefits of economic growth have been cornered by a few sections such as

rich and middle class. The impact of economic growth on poor has been little. As a result, the emphasis

on inclusive growth is necessary.

Approaches to Attain Inclusive Growth

Trickle down approach

The trickle down approach is a term used to describe the belief that if high income earners gain an increase in their income, their increased income and wealth benefits various sections of the society. For instance, if a person earns more, then he spends more. His expenses are income of other persons.

Page 40: Economics - PrepMate

Economic Growth and Development 11

Direct attack on poverty through government schemes

Trickle down approach has not been completely successful in attainment of inclusive growth. Benefits of

economic growth have overlooked poor sections of the society. The government has launched various

schemes for the welfare of particularly the poor section of the society. This approach is more effective to

bring inclusive growth because it is specifically targeted towards poor people.

11 LORENZ CURVE

The Lorenz curve is a graphical representation of wealth distribution developed by American

economist Max Lorenz in 1905. On the graph, the straight diagonal line represents perfect equality

of wealth distribution; the Lorenz curve lies beneath it, showing the reality of wealth distribution.

The difference between the straight line and the curved line is the amount of inequality of wealth

distribution.

Gini Coefficient

The Gini coefficient is calculated as follows:

Gini coefficient = Area between Line of Equality and Lorenz Curve

Area under Line of Equallity

The value of Gini coefficient ranges from 0 to 1. Tracking the Gini coefficient can demonstrate wealth trends in particular nations over time.

Lorenz Curve, Gini Coefficient, and Measuring Inequality

In an economy with perfect equality, 20% of the population would hold 20% of the wealth. As the

percentage of the population in consideration rises, so does their cumulative wealth.

Page 41: Economics - PrepMate

Economics12

In a perfect inequality curve, the Gini coefficient is 1, and the curve represents 100% of a nation’s wealth being held by one single person or entity.

The greater the disparity within a nation, the closer the Gini coefficient will be towards 1. The Gini coefficient of India for 2011 was 0.35.

12 TERMS DENOTING LEVEL OF ECONOMIC ACTIVITY

1. Slow down: Refers to reduction in the growth rate of the economy. For instance, let us suppose that

the growth rate of the economy has reduced from 8% to 5%. In the case of slowdown, economy

continues to grow but at a low rate.

2. Recession: Refers that the economy is reducing in size. Economists all over the world agree that recession means negative growth rate for a minimum of two consecutive quarters.

3. Expansion of the economy: Refers to increase in the size of the economy. In other words, expansion refers to a positive growth rate of economy. Expansion of economy and slowdown may occur at the same time.

4. Boom: Refers to increase in the growth rate of the economy. For instance, let us suppose that the growth rate of the economy has increased from 5% to 8%. In the case of boom, economy grows at a faster rate than the rate at which it grew previously.

5. Meltdown: A situation in which the economy of a country experiences a sudden downturn. An economy facing meltdown will most likely experience a falling GDP, drying up of liquidity, and rising/falling prices.

13 POVERTY ESTIMATES IN INDIA

In 1993, the Planning Commission constituted a task force for the calculation of poverty estimates in

India, chaired by Lakdawala. The committee made the following suggestions:

1. Consumption expenditure to be calculated based on calorie consumption, i.e. 2400 in rural areas and

2100 in urban areas.

Page 42: Economics - PrepMate

Economic Growth and Development 13

2. State-specific poverty lines should be constructed and updated using the Consumer Price Index of

industrial workers in urban areas and Consumer Price Index of agricultural labourers in rural areas.

Tendulkar Committee, 2005 (Headed by Suresh Tendulkar)

The Tendulkar Committee was constituted by the Planning Commission to address three shortcomings

in the previous methods of calculating poverty:

1. Consumption expenditure was linked to the consumption patterns of 1973–74. However, there have been significant changes in consumption patterns since 1973–74. Therefore, the Tendulkar Committee used the recent patterns of consumption to determine consumption expenditure.

2. Earlier, the expenditure on health and education was ignored. The Tendulkar Committee considers

expenditure on education as incurred by the state, but it considers health expenditure of `30 per

annum to seek health cover of `30,000.

3. Earlier, poverty estimates were based on uniform reference period, i.e. respondents were asked to detail the consumption made by them over the past 30 days. Whereas under the mixed reference period method, five low-frequency items such as clothing, footwear, durables, education expenditure, and health expenditure are surveyed over 365 days and all other items for previous 30 days.

The below poverty line population on the basis of the Tendulkar Committee report for 2011–12 was

25.7% rural population, 13.7% urban population, and 21.9% overall population.

Rangarajan Committee

In 2012, the Rangarajan Committee was established by the Planning Commission to review the Tendulkar Committee report. The new committee suggested further improvements in the poverty estimation methodology.

Tendulkar Rangarajan

Only counts expenditure on food, health, education, clothing

Food + non-food items such as education, healthcare, clothing, transport (conveyance), rent

— ∑ Urban poverty increased at a faster rate (40%) than rural poverty (19%).

∑ This is obvious, because Rangarajan included non-food items like rent, education, etc.

∑ These items/services are more expensive in cities than in villages.

— Rangarajan recommended that at any given point of time, the bottom 35% rural people and the bottom 25% urban people would always be considered poor.

— ∑ Poverty ratios should be disengaged from entitlements under government schemes.

∑ For example, cheap foodgrain quota under the Food Security Act should not be based on BPL-criteria, but social–caste census.

Page 43: Economics - PrepMate

Economics14

14 UNEMPLOYMENT

Unemployment is a phenomenon that occurs when a person who is actively searching for employment is unable to find work. Unemployment is often used as a measure of the health of the economy. The most frequently used measure of unemployment is the unemployment rate, which is the number of unemployed people divided by the number of people in the labour force.

While the definition of unemployment is clear, economists divide unemployment into many different categories.

Types of Unemployment

Disequilibrium unemployment: At the equilibrium level, the wage rate is such that the demand for labour matches with the supply of labour.

When the wage rates are pushed above the equilibrium levels due to the government-imposed minimum wage requirements or interference of trade unions, the demand for labour reduces and the supply for labour increases. This leads to a state of disequilibrium or mismatch between the demand and supply of labour.

This type of unemployment is also called classical or real wage unemployment.

Frictional unemployment or search unemployment: Frictional unemployment is the unemployment

that results from the time spent between jobs when a worker is searching for or transitioning from one

job to another. It is also called search unemployment.

Structural unemployment: Structural unemployment is a long-lasting form of unemployment caused

by fundamental shifts in an economy and exacerbated by extraneous factors such as technology,

competition, and government policy. Reasons why structural unemployment occurs include workers’

lack of requisite job skills or that workers live too far from regions where jobs are available and cannot

move closer. Jobs are available, but there is a serious mismatch between what employers need and what

workers can offer.

Page 44: Economics - PrepMate

Economic Growth and Development 15

Cyclical unemployment: Cyclical unemployment comes around due to the business cycle itself. Cyclical

unemployment rises during recessionary periods and declines during periods of economic growth.

Seasonal unemployment: It is a type of unemployment that is expected to occur at certain parts of the

year. For instance, amusement parks may experience seasonal unemployment because during months of

summer less people visit these parks.

Disguised unemployment: Disguised unemployment exists where part of the labour force is either left

without work or is working in a redundant manner, where marginal worker productivity is essentially zero.

In other words, disguised unemployment is a kind of unemployment in which some people look like being

employed but are actually not employed fully. Disguised unemployment is high in Indian agriculture.

Underemployment: A situation in which a worker is employed but not in the desired capacity, whether in

terms of compensation, hours, or level of skill and experience. The underemployed are often unsatisfied

and continue to look for suitable jobs.

Calculation of Unemployment

The unemployment rate is calculated by the following formula:

Unemployment rate =Number of unemployed

Total workforce

   

 ×100

In 2017, the unemployment rate in India was around 3.5%. The total workforce in India is above 30 crores and the number of unemployed are around 45 lakhs.

15 PHILLIPS CURVE

The Phillips curve is an economic concept developed by A. W. Phillips. This curve shows that inflation and unemployment have a stable and inverse relationship. The theory states that inflation comes with economic growth, which in turn should lead to more jobs and less unemployment.

However, the original concept has been somewhat disproven empirically due to the occurrence of stagflation, when there is high levels of both inflation and unemployment.

Page 45: Economics - PrepMate

Economics16

16 HUMAN RESOURCE DEVELOPMENT

Human resource development refers to improvement in the quality of life, increase in the number of opportunities, and in the freedom people enjoy.

The concept of human development, as understood today, was introduced by Dr. Mahbub ul Haq, a Pakistani economist. According to him, human development refers to an enlargement of people’s choices. Indian economist Prof. Amartya Sen has also adopted this view of human development.

The choices of people can be increased either by skill development of people (by granting vocational education), improving their health, or increasing their income level.

Education enhances choices with people as it confers them freedom to think and choose out of various career options. Health enlarges choices of people as a healthy person can undertake various types of activities in comparison to an unhealthy person. Increase in income level enlarges the options as it enables the people to buy various kinds of goods and services.

17 HUMAN DEVELOPMENT INDEX

Based on the above view, Human Development Index (HDI) was developed by Mahbub ul Haq along with Amartya Sen and was launched by the United Nations Development Programme (UNDP).

New Method for Calculating HDI

In its 2010 Human Development Report, the UNDP began using a new method for calculating the HDI. The following three indices are used:

1. A long and healthy life: Life expectancy at birth

2. Education index: Mean years of schooling and expected years of schooling

3. A decent standard of living: per capita income (PPP in US$)

Life Expectancy Index

Life Expectancy Index (LEI) is 1 when life expectancy at birth is 85 and 0 when life expectancy at birth is 20.

LEI = LE - 20

85 20-

LE: Life expectancy at birth in a given country

Education Index

Education Index (EI) =MYSI EYSI+

2

� Mean Years of Schooling Index (MYSI) = MYS

15

Fifteen is the maximum years of schooling.

Page 46: Economics - PrepMate

Economic Growth and Development 17

MYS: Mean years of schooling (i.e., years that a person aged 25 or older has spent in formal education)

� Expected Years of Schooling Index (EYSI) = EYS

18

Eighteen is equivalent to achieving a master’s degree in most countries.

EYS: Expected years of schooling (i.e., total expected years of schooling for children under 18 years of age)

Income Index

Income Index (II) is 1 when GNI per capita is $75,000 per year and 0 when GNI per capita is $100 per year.

II = GNI per capita -

-

100

75000 100

Value of HDI

The maximum value of HDI can be 1. Finally, the HDI is the geometric mean of Life Expectancy Index, Education Index, and Income Index.

HDI = LEI EI II3 × ×

Characteristics of nations with different levels of human development

Level of human development

High (0.7 and above) Medium (0.55 to 0.699) Low (0 to 0.549)

Nations North America, Northern and Western Europe

South America andAsia

Africa

State of economic development

Developed Developing Underdeveloped

Political condition Political stability

History of past instability but presently stable nations

Going through civil wars

18 MULTIDIMENSIONAL POVERTY INDEX

The global Multidimensional Poverty Index (MPI) was developed in 2010 by the Oxford Poverty and Human Development Initiative (OPHI) and the UNDP. It replaced the previous Human Poverty Index.

The global MPI is an international measure of acute poverty covering over 100 developing countries. It complements traditional income-based poverty measures by capturing the severe deprivations that each person faces at the same time with respect to education, health, and living standards.

The index uses the same three dimensions as the HDI: health, education, and standard of living. These are measured using 10 indicators.

Page 47: Economics - PrepMate

Economics18

Dimension Indicators

Health ∑ Child mortality ∑ Nutrition

Education ∑ Years of schooling ∑ School attendance

Living standards ∑ Cooking fuel ∑ Toilet ∑ Water ∑ Electricity ∑ Floor ∑ Assets

Indicators

The following 10 indicators are used to calculate the MPI:

Education (each indicator is weighted equally at 1/6)

1. Years of schooling: deprived if no household member has completed 6 years of schooling

2. Child school attendance: deprived if any school-aged child is not attending school up to class 8

Health (each indicator is weighted equally at 1/6)

3. Child mortality: deprived if any child has died in the family in the past 5 years

4. Nutrition: deprived if any adult or child for whom there is nutritional information is stunted

Standard of living (each indicator is weighted equally at 1/18)

5. Electricity: deprived if the household has no electricity

6. Sanitation: deprived if the household’s sanitation facility is not improved (according to Millennium

Development Goals (MDG) guidelines), or it is improved but shared with other households

7. Drinking water: deprived if the household does not have access to safe drinking water (according to

MDG guidelines) or safe drinking water is more than a 30-minute walk (roundtrip) from home

8. Floor: deprived if the household has a dirt, sand, or dung floor

9. Cooking fuel: deprived if the household cooks with dung, wood, or charcoal

10. Assets ownership: deprived if the household does not own more than one out of radio, TV, telephone,

bike, motorbike, or refrigerator and does not own a car or truck

A person is considered poor if they are deprived in at least a third of the weighted indicators. The

intensity of poverty denotes the proportion of indicators in which they are deprived.

Calculation of the Index

The MPI is calculated as follows:

MPI = H ¥ A

H: Percentage of people who are MPI poor (incidence of poverty)

A: Average intensity of MPI

Page 48: Economics - PrepMate

Economic Growth and Development 19

Practice Questions

1. An increase in national income because of an increase in prices only is called an

(a) Increase in real national income (b) Increase in national income at

constant prices(c) Increase in nominal national income(d) Increase in national income at base

year prices

2. The national income of a country for a given period is equal to the

(a) Consumption and investment expenditure incurred by the government

(b) Consumption and investment expenditure incurred by Indians abroad

(c) Consumption and investment expenditure incurred by private sector

(d) Consumption and investment expenditure incurred by all the above

3. In an open economy, the national income (Y) of the economy is (C, I, G, X, M stand for consumption, investment, government expenditure, total exports, and total imports, respectively.)

(a) Y = C + I + G + X(b) Y = C + I + G − X + M(c) Y = C + I + G + (X − M)(d) Y = C + I − G + X − M

4. The growth rate in per capita income at current prices is higher than that of per

capita income at constant prices, because the former takes into account the rate of

(a) Growth of population(b) Increase in price level(c) Growth of money supply(d) Increase in the wage rate

5. The value of all final goods and services produced by the normal residents of a country and their property, whether operating within the domestic territory of the country or outside, in a year is termed

(a) Gross national income(b) Net national income(c) Gross domestic product(d) Net domestic product

6. National income in context of India is the collective income of all the Indians. Which of the following is equivalent to national income?

(a) Gross domestic product at market prices

(b) Net domestic product at factor prices

(c) Net national product at market prices

(d) Net national product at factor cost

7. Gross domestic product is more than the net domestic product. Gross domestic product (GDP) is called “gross” because its computation does not exclude

(a) Consumption of capital in production process

(b) Subsidies on consumption of goods

Page 49: Economics - PrepMate

Economics20

(c) Earnings of foreign exchange in domestic country

(d) Informal economy

8. Which one of the following is responsible for bringing out the report on national and per capita income in India?

(a) Ministry of Planning(b) Ministry of Human Resource

Development(c) Ministry of Home Affairs(d) Ministry of Statistics and Programme

Implementation

9. The main reason for the low growth rate in India in spite of high rates of savings and capital formation is

(a) Low rate of investment (b) Low level of foreign investment(c) Low capital/output ratio(d) High capital/output ratio

10. In India, National Income Statistics are computed by which of the following?

(a) Planning Commission(b) Ministry of Finance(c) Central Statistical Office(d) RBI

11. Inclusive growth is the agenda of economic development in the future. The strategy of inclusive growth does not focus on

(a) Enhancement of education opportunities

(b) Reduction of poverty among backward castes

(c) Diversifying livelihood for tribal population

(d) Promotion of ancillary industry in industrial regions

12. Disguised unemployment typically leads

to low level of income levels despite

creating perception of high levels of

employment. Disguised unemployment

generally exists in

(a) Manufacturing sector

(b) Agriculture

(c) Small-scale industries

(d) Urban societies

13. When large number of workers loose

employment due to automation of

industrial processes, it is called

(a) Seasonal unemployment

(b) Structural unemployment

(c) Disguised unemployment

(d) Cyclical unemployment

14. The mechanization and automation of

economy is leading to high unemployment

rate in the economy. Which one of the

following is the unemployment rate?

(a) Percentage of unemployed to total

population

(b) Percentage of employed to total

labour force

(c) Percentage of unemployed to total

workforce

(d) Percentage of unemployed to total

persons not in labour force

15. If the net national product at market

price is higher than the net national

product at factor cost, then which of the

following is the correct description of

relationship between indirect taxes and

government subsidies:

(a) Government subsidies > Indirect

taxes

Page 50: Economics - PrepMate

Economic Growth and Development 21

(b) Government subsidies < Indirect taxes

(c) Government subsidies = Indirect taxes

(d) Government subsides ≥ Indirect taxes

16. Disguised unemployment generally means

(a) Total productivity of labour is zero(b) Average productivity of labour is

zero(c) Marginal productivity of labour is

zero(d) None of the above

17. Human Development Index comprises literacy rates, life expectancy at birth, and

(a) Gross domestic product per head in US dollars

(b) Gross domestic product per head at real purchasing power

(c) Gross national product in US dollars

(d) National income per head in US dollars

18. Which one of the following is the difference between value at market prices and value at factor cost for a national income aggregate?

(a) Value of consumption of fixed capital

(b) Value of indirect taxes net of subsidies

(c) Value of net factor income earned abroad

(d) Value of subsidies net of indirect taxes

19. Which one of the following is shown by the Phillips curve?

(a) Inverse relationship between real and nominal wages

(b) Inverse relationship between the rate of inflation and rate of unemployment

(c) Positive relationship between nominal wages and rate of employment

(d) Positive relationship between rate of inflation and nominal wages

20. The Lorenz curve is used to measure the level of inequality in an economy. It shows the relationship between

(a) Asset creation and income generation

(b) Population groups and their respective income shares

(c) Unemployment and inflation(d) Wage, labour hours, and leisure

21. The costs of factors of production, supplied by the entrepreneur himself, are known as

(a) Implicit costs(b) Explicit costs(c) Fixed costs(d) Variable costs

� Note: Implicit cost, also called an imputed cost, implied cost, or notional cost, is the opportunity cost equal to what a firm must give up in order to use a factor of production which it already owns and thus, for which it does not require to make any external payment. It is the opposite of an explicit cost, which is the payment made to outsiders.

Page 51: Economics - PrepMate

Economics22

22. The growth process that is sustainable and yields broad-based benefits and ensures equality of opportunity for all is termed

(a) Faster growth(b) Inclusive growth(c) Balanced growth(d) Sustained growth

23. In national income accounting, which one of the following is the difference between gross aggregates and net aggregates?

(a) The value of net indirect taxes(b) The value of consumption of fixed

capital(c) The value of intermediate

consumption(d) The value of final consumption

expenditure

24. The average rate of domestic savings (gross) for the Indian economy is currently estimated to be in the range of

(a) 15 to 20% (b) 20 to 25% (c) 25 to 30% (d) 30 to 35%

25. Which one among the following is the most appropriate definition of a country low on human development?

(a) A country with a small agricultural sector, low rate of population growth, and low per capita GNP

(b) A country with a high population growth rate, a high per capita GNP growth rate, and an unequal distribution of income

(c) A country with inadequate health care, poor educational facilities, and low per capita GNP

(d) A country with low per capita GNP, a large population, and a small agricultural sector

26. Consider the following statements:

1. The GDP of India (in nominal

terms) has crossed $2 trillion.

2. The per capita income (in nominal

terms) in India is more than

`100,000.

3. India has the third largest GDP in

the world in PPP terms.

4. The ICOR in India is lower than that

in the United States.

Which of the above statements are

correct?

(a) 1 and 2 only

(b) 2 and 3 only

(c) 2, 3, and 4 only

(d) 1, 2, and 3

27. Consider the following statements

regarding seasonal unemployment:

1. It results in large-scale migration

of agricultural labourers from

agriculturally backward regions to

that of urban developed regions.

2. Many agricultural workers especially

landless labourers and marginal

farmers remain unemployed after

employment season.

Which of the statements given above is/

are correct?

(a) 1 only (b) 2 only

(c) Both 1 and 2 (d) Neither 1 nor 2

28. Consider the following statements about

the Multidimensional Poverty Index:

1. It indicates the share of the popula-

tion that is multidimensionally

poor adjusted by the intensity

of deprivation in terms of living

standards, health, and education.

Page 52: Economics - PrepMate

Economic Growth and Development 23

2. In South Asia, only Afghanistan has higher levels of poverty than India.

Which of the statements given above is/are correct?(a) 1 only (b) 2 only(c) Both 1 and 2 (d) Neither 1 nor 2

� Note: The MPI placed Afghanistan as the poorest country in South Asia preceded by India, Bangladesh, Pakistan, Nepal, Bhutan, Sri Lanka, and Maldives, respectively.

29. Which among the following is added to the gross domestic product to arrive at national income of a nation?

1. Depreciation2. Subsidies3. Indirect taxes4. Net factor income abroadSelect the correct answer using the codes given below:(a) 2 only (b) 1 and 2 (c) 1 and 3 (d) 3 and 4

30. Consider the following statements:

1. The net domestic product can be greater than the gross domestic product for the same year.

2. The gross national product of an economy is always less than its gross domestic product for the same year.

Which of the statements given above is/are correct?(a) 1 only (b) 2 only(c) Both 1 and 2 (d) Neither 1 nor 2

31. Consider the following statements:

1. The income method of GDP calculation considers income received by factors of production only.

2. The GDP calculated through income method is less than the GDP calculated through the expenditure method for the same year.

Which of the statements given above is/are correct?(a) 1 only (b) 2 only(c) Both 1 and 2 (d) Neither 1 nor 2

32. With reference to national income accounting, consider the following statements:

1. The value of GVA is lower than GDP because the taxes levied are higher than the subsidies granted.

2. GVA gives measure from the consumer’s point of view and GDP gives measure from producer’s point of view.

Which of the statements given above is/are correct?(a) 1 only (b) 2 only(c) Both 1 and 2 (d) Neither 1 nor 2

33. Consider the following pairs:

Type of unemployment Example

1. Structural unemployment An unemployed worker from cycle industry because people are buying more cars

2. Cyclical This kind of unemployment occurs when all those who want to work cannot be employed because there is not enough demand in the market for their work.

3. Frictional unemployment A person leaving his/her job to prepare for civil services.

Page 53: Economics - PrepMate

Economics24

Which of the pairs given above is/are

correctly matched?

(a) 1 and 2 only (b) 2 and 3 only

(c) 3 only (d) 1, 2, and 3

34. Consider the following statements:

1. The nominal gross domestic

product (GDP) reflects the changes

in both prices and quantities of the

goods and services produced in an

economy.

2. Real GDP reflects only the changes

in the quantities of the goods and

services produced in an economy.

3. GDP deflator reflects the changes

in prices of the goods and services

produced in an economy.

Which of the statements given above is/

are correct?

(a) 1 and 2 only (b) 1 and 3 only

(c) 2 and 3 only (d) 1, 2, and 3

�Note: In economics, the GDP deflator

is a measure of the level of prices in an

economy. It is calculated by dividing

nominal GDP by real GDP and then

multiplying by 100. Nominal GDP is

the market value of goods and services

produced in an economy, unadjusted for

inflation.

35. GDP is defined as the output of economic

activities carried out within the economic

boundaries of a country. Which of the

following constitutes the economic

territory of a country with respect to

GDP calculation?

1. Fishing vessels, oil and natural gas

rigs operated by the residents of a

country in the international waters.

2. Embassies, consulates, and military establishment of the country located abroad.

3. Ships and aircraft owned by residents of the country operating between two different countries.

4. Industrial establishment owned by residents located abroad.

Select the correct answer using the codes given below:(a) 2 and 3 only (b) 1 and 3 only(c) 1, 2, and 3 (d) 1, 2, 3, and 4

36. With reference to assessment of labour market condition, consider the following statements:

1. The labour force participation rate is the ratio of the number of persons in the labour force to the total population.

2. The unemployment rate is the ratio of the number of unemployed persons to the total population.

Which of the statements given above is/are correct?(a) 1 only (b) 2 only(c) Both 1 and 2 (d) Neither 1 nor 2

37. Which of the following affect the accuracy of national income estimation in India?

1. Output of non-monetized sector2. Non-availability of data about

income of small producers or household enterprises

3. Unreported legal income4. InflationSelect the correct answer using the codes given below:(a) 1, 2, 3, and 4 (b) 1, 2, and 3 only(c) 2 and 3 only (d) 1 and 4 only

Page 54: Economics - PrepMate

Economic Growth and Development 25

1. In the context of Indian economy, consider the following pairs: (2010)

Term Most Appropriate

description

1. Meltdown Fall in stock prices

2. Recession Fall in growth rate

3. Slow down Fall in GDP

Which of the pairs given above is/are correctly matched?(a) 1 only (b) 2 and 3 only(c) 1 and 3 only (d) 1, 2, and 3

2. With reference to Indian economy, consider the following statements: (2010)

1. The gross domestic product (GDP) has increased by four times in the last 10 years.

2. The percentage share of public sector in GDP has declined in the last 10 years.

Which of the statements given above is/are correct?(a) 1 only (b) 2 only(c) Both 1 and 2 (d) Neither 1 nor 2

3. In the context of Indian economy, consider the following statements: (2011)

1. The growth rate of GDP has steadily increased in the last 5 years.

2. The growth rate in per capita income has steadily increased in the last 5 years.

Which of the statements given above is/are correct?(a) 1 only (b) 2 only(c) Both 1 and 2 (d) Neither 1 nor 2

4. The Multidimensional Poverty Index developed by the Oxford Poverty and Human Development Initiative with UNDP support covers which of the following? (2012)

1. Deprivation of education, health, assets, and services at household level

2. Purchasing power parity at national level

3. Extent of budget deficit and GDP growth rate at national level

38. Which of the following statements is/are correct?A rise in per capita income does not automatically imply an increase in economic welfare because1. Distribution of income is not known2. Rate of economic growth is not

known

3. Per capita income suffers from the limitation of averages

Select the correct answer using the codes given below:(a) 1 only(b) 2 only(c) 1 and 3 only(d) 1, 2, and 3

PERFECTING PAST PRELIMS

Page 55: Economics - PrepMate

Economics26

Select the correct answer using the codes given below:(a) 1 only(b) 2 and 3 only(c) 1 and 3 only(d) 1, 2, and 3

5. Economic growth in country X will necessarily have to occur if (2013)

(a) There is technical progress in the world economy

(b) There is population growth in X(c) There is capital formation in X(d) The volume of trade grows in the

world economy

6. The national income of a country for a given period is equal to the (2013)

(a) Total value of goods and services produced by the nationals

(b) Sum of total consumption and investment expenditure

(c) Sum of personal income of all individuals

(d) Money value of final goods and services produced

7. Disguised unemployment generally means (2013)

(a) Large number of people remain unemployed

(b) Alternative employment is not available

(c) Marginal productivity of labour is zero

(d) Productivity of workers is low

8. With reference to Indian economy, consider the following statements: (2015)

1. The rate of growth of real gross domestic product has steadily increased in the last decade.

2. The gross domestic product at market prices (in rupees) has steadily increased in the last decade.

Which of the statements given above is/are correct?(a) 1 only (b) 2 only(c) Both 1 and 2 (d) Neither 1 nor 2

9. Increase in absolute and per capita real GNP do not connote a higher level of economic development, if (2018)

(a) Industrial output failed to keep pace with agricultural output.

(b) Agricultural output fails to keep pace with industrial output.

(c) Poverty and unemployment increase.

(d) Imports grow faster than exports.

10. Consider the following statements: (2018)

Human capital formation as a concept is better explained in terms of a process which enables 1. Individuals of a country to

accumulate more capital. 2. Increasing the knowledge, skill

levels and capacities of the people of the country.

3. Accumulation of tangible wealth.4. Accumulation of intangible wealth. Which of the statements given above is/are correct?

Page 56: Economics - PrepMate

Economic Growth and Development 27

; ANSWER KEYS

Practice Questions

1. (c) 2. (d) 3. (c) 4. (b) 5. (a)

6. (d) 7. (a) 8. (d) 9. (d) 10. (c)

11. (d) 12. (b) 13. (b) 14. (c) 15. (b)

16. (c) 17. (d) 18. (b) 19. (b) 20. (b)

21. (a) 22. (b) 23. (b) 24. (d) 25. (c)

26. (d) 27. (c) 28. (c) 29. (a) 30. (d)

31. (a) 32. (a) 33. (d) 34. (d) 35. (c)

36. (d) 37. (a) 38. (c)

Perfecting Past Prelims

1. (a) 2. (b) 3. (d) 4. (a) 5. (c)

6. (b) 7. (c) 8. (b) 9. (c) 10. (b)

11. (d) 12. (d)

(a) 1 and 2 (b) 2 only (c) 2 and 4 (d) 1, 3 and 4

11. Despite being a high saving economy, capital formation may not result in significant increase in output due to (2018)

(a) Weak administrative machinery (b) Illiteracy (c High population density (d) High capital-output ratio

12. If a commodity is provided free to the public by the government, Then (2018)

(a) The opportunity cost is zero. (b) The opportunity cost is ignored.(c) The opportunity cost is transferred

from the consumer of the product to the tax-paying public.

(d) The opportunity cost is transferred from the consumers of the product to the Government.

Page 57: Economics - PrepMate

Solutions for

Practice

Questions and

Perfecting Past

Prelims

Page 58: Economics - PrepMate
Page 59: Economics - PrepMate

393Solutions

CHAPTER 1 ECONOMIC GROWTH

AND DEVELOPMENT

Practice Problems

1. (c) Nominal national income can increase on account of either increase in prices or increase in production or both. Thus, increase in national income due to increase in prices only is an example of increase in the nominal national income.

2. (d) Income is sum total of investment and consumption. The income belongs to private sector in India, Indians abroad and the government.

3. (c) In economics, aggregate expenditure (AE) is a measure of national income. The aggregate expenditure is thus the sum total of all the expenditures undertaken in the economy by the factors during a given time period. It refers to the expenditure incurred on consumer goods, investment and the expenditure made by the government in the economy. In an open economy scenario, the aggregate expenditure also includes the difference between the exports and the imports.

4. (b) Growth rate at constant prices is calculated on the basis of prices of goods at base year, so it can only increase when output of goods and services in particular year increases.

Growth rate at current price is calculated on the basis of prices of goods and services in the same year in which they are produced. Even if there is no increase in physical output, the growth rate may increase due to rise in price levels.

5. (a) The value of all final goods and services produced by the normal residents of a country and their property, whether operating within the domestic territory of the country or outside in a year is termed as Gross National Income.

7. (a) Gross Domestic Product (GDP) is more than Net Domestic product (NDP) because GDP does not take into account the depreciation of assets.

NDP = GDP − Depreciation

8. (d) The reports on per-capita income and National Income are brought out by Central Statistical Office (CSO) under the Ministry of Statistics and Program Implementation.

9. (d) High ICOR is responsible for low growth rate in India despite a high rate of saving and capital formation. The higher the ICOR, the lower the output generated from use of capital.

10. (c) The Central Statistics Office (CSO), Ministry of Statistics and Programme Implementation, releases National Income Statistics.

11. (d) Inclusive growth is a concept that advances equitable opportunities for economic participants during economic growth with benefits incurred by every section of society. Industrial regions are already receiving large benefits of economic growth and are particularly well of than other sections in the economy.

12. (b) There is a high level of disguised unemployment in Indian agriculture.

13. (b) Rise in unemployment levels due to change in technology is called Structural unemployment.

Page 60: Economics - PrepMate

394 Economics

15. (b) If Net National Product at Market price is higher than Net National Product at Factor cost, it means that government subsidies are less than indirect taxes.

Market Price – Indirect Taxes + Subsidies = Factor Cost

17. (d) Human development index is calculated on the basis of:

(i) Life expectancy at birth

(ii) Literacy Rate

(iii) Per capita income (PPP US$)

18. (b) Indirect Taxes – Subsidies = NNPmp

– NNP

fc

19. (b) The Phillips Curve shows inverse relationship between the rate of inflation and rate of unemployment.

20. (b) The Lorenz Curve is used to measure the level of inequality in an economy. The Lorenz Curve shows the relationship between population groups and their respective income shares.

22. (b) The growth process that is sustainable and yields broad-based benefits and ensures equality of opportunity for all is termed as Inclusive growth.

25. (c) A country low on human development performs poorly on the indicators used for determining human development. Such poor performance is due to inadequate health care, poor educational facilities, and low per capita GNP.

26. (d) Statement 1 is correct: Nominal GDP of India is a $2.26 trillion.

Statement 2 is correct. India’s per capita income grew by 9.7 percent to `1,03,219 in 2016-2017 from `94,130.

Statement 3 is correct. India has third largest GDP in purchasing power parity terms ($8.72 trillion as of 2016) after China and USA.

Statement 4 is incorrect. ICOR of USA is around 3 while India’s ICOR is near 4.

29. (a) Gross domestic product + Subsidies – Indirect taxes – Net factor income abroad = National income of a nation.

30. (d) Statement 1 is incorrect. NDP = GDP − Depreciation.

Statement 2 is incorrect. GNP = GDP − Factor income paid to foreigners in India + Factor income paid to Indians abroad.

GNP depends on two factors. If the factor income paid to foreigners in India is greater than factor income paid to Indians abroad, then the GNP is less than GDP and vice versa. Thus, GNP is not always less than GDP.

31. (a) Statement 2 is incorrect. The value of GDP is same irrespective of the method used to calculate GDP.

33. (d) Structural unemployment is caused by fundamental shifts in economy and exacerbated by extraneous factor such as technology, competition, etc.

Cyclical unemployment rises during recessionary periods and declines during periods of economic growth.

Frictional unemployment rises when worker is searching for better job.

34. (d) Statement 1 is correct. Nominal GDP is calculated on the basis of current prices of particular year in which goods and services are produced.

Page 61: Economics - PrepMate

395Solutions

Statement 2 is correct. Real gross domestic product is calculated on the basis of prices of the base year. It can increase when the quantity of goods and services produced in a particular year increases.

35. (c) The term economic territory includes

• The geographic territory administered by a government within which persons, goods, services, and capital move freely;

• Any free zones, including bonded warehouses and factories under customs control;

• The national air-space, territorial waters, and the continental shelf lying in international waters, over which the country enjoys exclusive rights;

• Territorial enclaves in the rest of the world under international treaties (embassies, consulates, military bases, scientific bases, etc.);

• Deposits of oil, natural gas, and other reserves in international waters owned by residents of a country.

The economic territory does not include extraterritorial enclaves (i.e., the parts of the country’s own geographic territory used by governments of other countries. For instance, foreign embassies and consulates within India are not part of India’s economic territory).

36. (d) Statement 1 is incorrect. Labour force participation rate is calculated by dividing the number of people actively participating in the labour force by the total number of people eligible to

participate (not total population) in the labour force. People who are not looking for a job such as full-time students, homemakers, etc., will not be a part of the data set.

Statement 2 is incorrect. Unemployment rate is the ratio of the number of unemployed persons to the total workforce.

37. (a) The accuracy of national income estimation is affected by all the mentioned factors because it is difficult to determine the exact value of these factors.

38. (c) Statement 2 is incorrect. A rise in per capita income signals growth in the economy. Statement 2 does not justify why rise in per capita income does not automatically imply an increase in economic welfare.

Statements 1 and 3 are correct. Per capita income shows only the average income. A nation may have a significant GDP, but the distribution of income may be confined to a few rich people and the remaining population may be living under poverty.

Perfecting Past Prelims

1. (a) Recession is generally identified by a fall in GDP in two successive quarters.

An economic slowdown occurs when the rate of economic growth slows in an economy.

2. (b) Statement 1 is incorrect. The growth rate of GDP in 1990–2000 was 4.4% and that in 2000–2010 was 6.4%. For GDP to quadruple, country has to grow by 10% for almost more than 15 years.

Page 62: Economics - PrepMate

396 Economics

Statement 2 is correct. The growth rate of private sector far exceeds that of public sector (government-owned enterprises). Thus, the percentage share of public sector in GDP has declined in the last 10 years.

3. (d) Statement 1 is incorrect. The question was asked in 2011. The statement was clearly incorrect because the growth rate had come down due to the financial crisis in 2008.

Statement 2 is incorrect. If the GDP growth rate is not steadily increasing, per capita income growth rate would also not be steadily increasing.

4. (a) Statements 2 and 3 are incorrect. Purchasing power parity at national level, extent of budget deficit, and GDP growth rate at national level are not covered by multidimensional poverty index.

5. (c) Economic growth depends upon two factors: capital formation and ICOR.

6. (b) Net national product at factor cost (NNPfc) or national income is the collective income earned by nationals of a country in a given time period. This income can be further bifurcated into consumption and investment expenditure.

7. (c) Disguised unemployment exists where extra work force is not contributing anything towards production. In other words, marginal productivity of extra labour is zero.

8. (b) Statement 1 is incorrect. The rate of growth of real gross domestic product (economic growth rate) has not increased steadily in the last decade.

Statement 2 is correct. GDP has been increasing year after year.

9. (c) Economic Development considers not just quantitative indicators such as GNP, GDP but also qualitative indictors such as standards of living, poverty, unemployment, etc.

10. (b) This question requires careful analysis. It talks about the process which explains Human Capital formation.

Statement 2 is correct. Human Capital formation is the process which enables increasing the knowledge, skill levels and capacities of the people of the country.

Many people tend to confuse Statement 4 as correct. However, accumulation of intangible wealth such as copyrights, patents, etc is outcome of Human capital formation and it is not the process. Thus, Statement 4 is incorrect.

Similarly, Statements 1 and 3 are also incorrect because they are also outcome of Human capital formation.

12. (d) Option (a) is incorrect. A free good is a good with zero opportunity cost. However, one important distinction is that just because a good is given away for ‘free’ it doesn’t necessarily mean it is a free good. In this case, if the government wasn’t giving away free commodity to the public, it might be using the same money elsewhere.

Option (b) is incorrect. Everything has a cost. Thus, Opportunity cost cannot be ignored.

Option (c) is incorrect. The tax receipts, once collected, belong to

Page 63: Economics - PrepMate

397Solutions

the government. Thus, their further use involves opportunity cost to the government.

Option (d) is the correct answer. Here, opportunity cost is what government gave up in order to provide free commodities. If the government would not have given free commodities, then it could have used the same funds in any other manner.

CHAPTER 2 TAXATION SYSTEM OF

INDIA

Practice Questions

3. (b) The Laffer curve was developed by economist Arthur Laffer. The curve explains the relationship between different tax rates and the amount of tax revenue collected by government.

5. (b) The redistribution of income can be best brought through levying higher taxes on people with higher incomes (progressive taxation) and spending larger amount of money on people with lower incomes (regressive expenditure).

10. (d) Sales tax (VAT) is levied, collected, and appropriated by the State governments. It has been replaced with GST.

13. (b) Statement 1 is incorrect. The responsibility for the administration of GST is divided between state governments and union government.

14. (c) Input credit is not available for goods or services used for personal consumption.

17. (b) Statement 1 is incorrect. Profits arising from sale of flats kept as stock by a property dealer are gains from business.

Statements 2 and 3 are correct. A flat is a capital asset for a salaried employee. Similarly, business premises are a capital asset for a business man. Thus, profits arising from their sale is capital gain.

18. (a) Statement 2 is incorrect. Tax is levied on people in proportion to the benefit they earn themselves.

Statement 3 is incorrect. A progressive taxation system has lesser impact on the receipts of poor people than that of rich people.

19. (c) Statement 1 is correct. Increase in formal sector of the economy brings more people under the tax umbrella, leading to a rise in tax to GDP ratio.

Statement 2 is correct. With rise in income level, people will be liable to pay more taxes due to progressive nature of taxation system. Rise in income levels will also lead to increase in expenditure levels, leading to enhanced indirect tax collection.

20. (a) Statement 1 is correct. The concept of MAT was introduced to tax companies making high profits and declaring dividends to their shareholders under Companies Act but have no significant taxable income under Income Tax Act because of exemptions, deductions, and incentives.

Remaining statements are not related to MAT.

Page 64: Economics - PrepMate

Introduction

to Writing

Answers for

Mains

Page 65: Economics - PrepMate
Page 66: Economics - PrepMate

431Introduction to Writing Answers for Mains

A good answer is based on the following aspects

Content analysis

Is your answer

addressing

the question?

Have you

attempted

all the subparts

of question?

Does your

answer require

an introduction?

Is there any

need to

underline?

Should you

adhere to

word limit?

Does your

answer require

a conclusion?

Should answers

be written

in points or

paragraph?

Should you cover

the complete space

given for attempting

answer?

Presentation

analysis

Page 67: Economics - PrepMate

432 Economics

Let us consider the aspects mentioned in the diagram on one-by-one basis.

1. Is Your Answer Addressing the Question?

The most common reason to blame for a low score is the lack of candidate’s ability to address the question.

Have you ever come across a candidate who attempted almost all (or all) the questions, yet could not clear the mains examination? If yes, you have probably met the candidate who wrote answers that did not address the questions.

To understand how to address a question appropriately, we can classify each question into two parts: ‘Statement’ and ‘Directive’.

For example:

The National Urban Transport Policy emphasizes on moving people instead of moving vehicles. Discuss critically the success of various strategies of the government in this regard. (UPSC Mains, 2014)

In this question, the Statement is “various strategies of the government in respect of National Urban Transport Policy, which emphasizes on moving people instead of moving vehicles” and the directive is “Discuss critically”.

Suppose a candidate “explains” the National Urban Transport Policy, which emphasizes on moving people instead of moving vehicles, or various strategies of the government in respect of the transport policy. The marks of such a candidate would be severely deducted.

Critical discussion of the question requires pros and cons of the subject matter. Thus, the question demands the pros and cons of various strategies of the government in

respect of the National Urban Transport Policy, which emphasizes on moving people instead of moving vehicles.

It is to be noted that a particular directive may be attached to a sub-part of a question. In a single question, there can be as many directives as there are number of sub-parts.

So what all directives

UPSC can use? What do

these directives mean?Here is a simple list for you. We will further learn about their application by attempting past year questions in the coming pages.

1. Enumerate: Mention a number of things one by one or give a list of things.

2. Highlight: Draw special attention to something.

3. Outline: Give a summary of something.

4. Discuss: Write about a topic in detail, taking into account different issues or ideas. For example,

Though India allowed foreign direct investment (FDI) in what is called multi-brand retail through joint venture route in September 2012, the FDI even after a year has not picked up. Discuss the reasons.

(UPSC Mains, 2013)

5. Explain: What is meant by a word, text, concept, or action. For example,

“While we flaunt India’s demographic dividend, we ignore the dropping rates of employability.” What are we missing while doing so? Where will the jobs that India desperately needs come from? Explain. (UPSC Mains, 2014)

Page 68: Economics - PrepMate

Previous Year’s Questions

(Mains) with

Solutions

Page 69: Economics - PrepMate
Page 70: Economics - PrepMate

439Previous Year’s Questions (Mains) with Solutions

1. With a consideration towards the strategy of inclusive growth, the new company’s bill, 2013, has indirectly made CSR a mandatory obligation. Discuss the challenges expected in its implementation in the right earnest. Also discuss other provisions in the bill and their implications. (UPSC Mains, 2013)

Sol. The question is based on the current affairs of the year 2013.

2. What are the reasons for introduction of Fiscal Responsibility and Budget Manage-ment (FRBM) Act, 2003? Discuss critically its salient features and their effectiveness.

(UPSC Mains, 2013)

Sol.

Analysis of Question

Discuss critically The question requires break-up of the issue or situation into various parts and thorough analysis of these parts.

Number of parts Three sub-partsPart I—Reasons for introduction of Fiscal Responsibility and Budget Management (FRBM) Act, 2003.Part II—Critical discussion on its salient features.Part III—Critical discussion on its effectiveness.

Mode of presentation Point form

Importance of conclusion

Required

Reasons for the introduction of Fiscal

Responsibility and Budget Management

Act, 2003

The FRBM Act, 2003 was adopted to bring

fiscal discipline at both the centre and state

levels. Specifically, the following reasons were

behind the adoption of FRBM Act, 2003:

1. Huge fiscal and revenue deficits were

posing question over the long-term

financial stability of India.

2. High level of deficit and debt levels raised

the cost of fresh borrowing.

3. India is a federal country. There was a need

that the central government should have

the means to keep a check on the financial

position of states.

Salient features of the act and rules under

the act

1. Every year, the government will bring

down the revenue deficit by 0.5% and

eliminate it by 2007–08.

2. Every year, the government will bring

down the fiscal deficit by 0.3% and bring

it down to 3% by 2007–08.

3. Total liabilities of the union government

should not rise by more than 9% a year.

4. The union government would not give

guarantee for loans raised by PSUs and

state governments exceeding 0.5% of the

GDP in aggregate.

5. State governments were asked to formulate

their own FRBM acts. Those states that

have formulated their own FRBM acts

have been given autonomy to borrow

further without the permission of the

central government.

Page 71: Economics - PrepMate

440 Economics

Criticism of the FRBM Act

1. Measures to enforce compliance:

It required the Finance Minister to

place quarterly reports of finance and

expenditure before the Parliament.

Deviations from targets are required to

be approved by the Parliament. No other

measures for the failure of compliance

have been specified.

2. The requirement to adhere financial

discipline may force the government to

cut back on the social expenditure.

3. Off-budget items such as oil bonds are

not counted in government expenditure.

4. Impact of global financial crisis: Due to

the 2008 global financial crisis, the

deadlines under the act were postponed

in 2009. After overcoming the 2008

global financial crisis, the FBRM Act was

readopted in 2012–13 but not in a strict

sense.

5. Need for flexibility: Recently our

Finance Minister has asked for a range

of targets under the FRBM Act rather

than a specific pinpointed figure in order

to provide flexibility in operations to the

government.

Conclusion

The FRBM Act has brought the concerns of

financial discipline to the forefront. However,

the act lacks the teeth to enforce its compli-

ance by union and the states.

3. What is meaning of the term tax expenditure?

Taking the housing sector as an example,

discuss how it influences budgetary policies

of the government. (UPSC Mains, 2013)

Sol.

Analysis of Question

Discuss Write about a topic in detail, taking into account different issues or ideas.

Number of parts

Three sub-partsPart I—Meaning of the term tax expenditure.Part II—Example of housing sector.Part III—Tax expenditure influence on budgetary policies

Mode of presentation

Mix of paragraph and point form

Importance of conclusion

Not required

Tax expenditure is the amount of tax foregone by the government in order to support or meet its policy targets. In simple words, the government can give a lot of concessions, reliefs, exemptions, and deductions in the tax, and by doing this, the government can promote industries or businesses, investments, etc.

Examples of tax expenditure in the housing sector

For example, in the housing sector, the government has a lot of policies that come in the bracket of “tax expenditure”. Some of the tax exemptions are as follows:

1. Capital gains tax is exempted if the amount is invested in a new house.

2. Exemptions from excise duty are often provided to the manufacturers of ready mix concrete (which is used for housing).

3. If one has taken a housing loan, one gets a deduction from income tax for repayment of the loan.